Вы находитесь на странице: 1из 9

1.

A carrier is frequency modulated by a sinusoidal modulating signal of frequency 2 KHz,


resulting in a frequency deviation of 5 KHz. What is the bandwidth occupied by the
modulated waveform? The amplitude of the modulating signal is increased by a factor of
3 and its frequency is lowered to 1 KHz. What is the new bandwidth?
2. The message signal m ( t )=10 sinc(4000 t) frequency modulates the carrier
c ( t ) =100 cos 2 π f c t .The modulation index is 6.
a. Write an expression for the modulated signal s ( t ) .
b. What is the maximum frequency deviation of the modulated signal?
c. What is the power content of the modulated signal?
d. Find the bandwidth of the modulated signal.
3. Signal m ( t ) is shown in Figure P-3.25. This signal is used once to frequency modulate a
carrier and once to phase modulate the same carrier.
a. Find the relation between k p and k f such that the maximum phase of the
modulated signals in both the cases are equal.
b. If k p = f d =1, what is the maximum instantaneous frequency in each case where f d
denotes the peak frequency deviation.

4. An angle modulated signal has the form


x ( t )=100 cos ¿ ¿ Where f c =10 MHz .
a. Determine the average transmitted power.
b. Determine the peak-phase deviation.
c. Determine the peak-frequency deviation.
d. Is this FM or PM signal? Explain.
5. The Armstrong type FM modulator is shown in figure P.28. The narrowband FM signal
has maximum angular deviation of 0.10 radians in order to keep the distortions under
control.
a. If the message signal has bandwidth of 15KHz and the output frequency from the
oscillator is 100KHz, determine the frequency multiplication that is necessary to
generate an FM signal at carrier frequency of f c =104 MHz and frequency
deviation of ∆ f =75 KHz .
b. If the carrier frequency for the wideband FM signal is to be within ± 2 Hz ,
determine the maximum allowable drift of 100KHz oscillator.

6. Consider a super heterodyne receiver designed to receive the frequency band of 1 to


30MHz with IF frequency 8 MHz. What is the range of frequencies generated by local
oscillator for this receiver? An incoming signal with carrier frequency 10MHz is received
at 10MHz setting. At this setting of the receiver we also get interference from other
carrier frequency signal if the receiver RF stage bandpass filter has poor selectivity. What
is the carrier frequency of interfering signal?
7. An angle Modulated Signal with carrier frequency ω c =2 π ×105 is described by the
equation φ EM (t )=10 cos ¿¿
a. Find the power of the modulated signal
b. Find the frequency deviation ∆ f .
c. Find the deviation ratio β .
d. Find the phase deviation ∆ φ .
e. Estimate the bandwidth of φ EM (t ) .
8. A 2 kHz audio signal modulates a 50 MHz carrier causing frequency deviation of 2.5 kHz .
Determine the Bandwidth of FM signal.
9. Design (only the block diagram) an Armstrong indirect FM modulator to generate an FM
carrier with carrier frequency of 98.1 MHz and ∆ f =75 kHz . A narrow-band FM
generator is available at a carrier frequency of 100kHz and frequency deviation
∆ f =10 Hz . The stock room also has an oscillator with an adjustable frequency in the
range of 10 to 11 MHz. There are also plenty of frequency doublers, triplers and
quintriplers.
10. Design (only the block diagram) an Armstrong indirect FM modulator to generate an FM
carrier with carrier frequency of 96MHz and ∆ f =20 kHz . A narrow band FM generator
with f c =200 kHz and adjustable ∆ f in the range of 9 to 10Hz is available. The stock
room also has an oscillator with adjustable frequency in the range of 9 to 10MHz. There
is bandpass filter with any center frequency and only frequency doublers are available
11. For a modulating signal
m ( t )=2cos 100 t +18 cos 2000 πt
a. Write the expression (do not sketch) for PM, φ PM (t) and FM, φ FM (t) when A=10,
ω c =106 rad , k f =1000 π , k p =1. For determining φ FM ( t ) , use the indefinite integral of
m ( t ) , that is, take the value of the integral at t=−∞ to be 0.
b. Estimate the bandwidths of φ PM (t) and φ FM (t).
12. An angle modulated signal with carrier frequency ω c =2 π ×10 6 is described by the
equation φ EM (t )=10 cos ¿¿ . Find the
a. Power of the angle modulated signal.
b. Frequency deviation ∆ f
c. Phase deviation ∆ φ .
d. Estimate the bandwidth of φ EM (t )
13. Determine the amplitude and phase of various frequency components of a PM signal with
k p=1 and m ( t ) a periodic signal given by
Tm
m ( t )=
{
−1 ,
1 ,0 ≤ t ≤
Tm
2
≤ t ≤ Tm

14. An FM signal is given by


2
in one period.

t
x ( t )=100 cos [2 π f c t+100 ∫ m ( τ ) dτ ] where m(t) is shown in figure P-3.30.
−∞

a. Sketch the instantaneous frequency as a function of time.


b. Determine the peak-frequency deviation.
15. The carrier c ( t ) =100 cos 2 π f c t is frequency modulated by message signal
m ( t )=5 cos 20000 πt, where f c =100 MHz. The peak frequency deviation is 20KHz.
a. Determine the amplitude and frequency of all the signal components that have a
power level of atleast 10% of the power of unmodulated carrier component.
b. From Carson’s rule, determine the Bandwidth of FM signal.
16. The carrier c ( t ) =A cos 2 π 106 t is angle modulated (PM or FM)by the sinusoid signal
rad Hz
m ( t )=2cos 2000 πt . The deviation constants are k p=1.5 ∧k f =3000 .
V V
a. Determine β FM ∧β PM .
b. Determine the bandwidth in each case using Carson’s rule.
c. Plot the spectrum of modulated signal in each case(plot only those frequency
components that lie within bandwidth derived in part b.
d. If the amplitude of m ( t ) is decreased by factor of 2, how would your answers to
part a-c change?
e. If the frequency of m ( t ) is increased by factor of 2, how would your answers to
part a-c change?
17. The carrier signal c ( t ) =100 cos 2 π 108 t is phase modulated by the signal
π
m ( t )=5 cos 2000 πt . The PM signal has the peak phase deviation of .
2
a. Determine the magnitude spectrum of sinusoid components and sketch the results.
b. Using Carson’s rule, determine the approximate bandwidth of the PM signal and
compare the results with the analytical result in part a.
18. An angle modulated signal has the form ( t )=100 cos ¿ ¿ .
a. Assuming that this is an FM signal, determine the modulation index and
transmitted signal bandwidth.
b. Repeat part a. if f m is doubled.
c. Assuming that this is an PM signal, determine the modulation index and
transmitted signal bandwidth.
d. Repeat part c. if f m is doubled.
19. An angle modulated signal is given by x ( t )=100 cos [2000 πt+ φ ( t ) ], where
a ¿ φ ( t )=5 cos 20 πt and b ¿ φ ( t )=5 sin 20 πt. Determine and sketch the amplitude and
phase spectra for part a) and b), compare the results.
Hz
20. The message signal m(t) into an FM modulator with peak frequency deviation f d =25
V
is shown in the figure P. 3.38.Plot the frequency deviation in Hz and phase deviation in
radians.

21. A message signal m(t) has bandwidth of 10KHz and peak amplitude |m ( t )| of 1V.
Estimate the bandwidth of the modulated signal obtained when m(t) frequency modulates
a carrier with peak frequency deviation of
Hz
a. f d =10
V
Hz
b. f d =100
V
Hz
c. f d =1000
V
22. The modulating signal into an FM modulator is m ( t )=10 cos 16 πt . The output of FM
modulator is x ( t )=10 cos ¿¿ where k f =10. If the output of FM modulator is passed
through an ideal BPF centered at f c =2000 Hz with bandwidth of 62Hz. Determine the
power of frequency components at the output of the filter. What percentage of transmitted
power appears at the output of the BPF.? Figure P.3.40.
23. The message signal m 1 (t) is shown in figure P.3.41 and the message signal
m2 ( t )=sinc ( 2 ×104 t ) , in volts again.
a. If m 1 (t) is frequency modulated with carrier of 1MHz with frequency deviation
Hz
constant k f =5 , what is the maximum instantaneous frequency of the
V
modulated signal?
b. If m 1 (t) is phase modulated with carrier of 1MHz with phase deviation constant
radians
k p=3 , what is the maximum instantaneous frequency of the modulated
V
signal? what is the minimum instantaneous frequency of the modulated signal?
c. If m 2 (t) is frequency modulated with carrier of 1MHz with frequency deviation
Hz
constant k f =1000 , what is the maximum instantaneous frequency of the
V
modulated signal? What is the bandwidth of the modulated signal.

24. The super heterodyne FM receiver operates in the range of 88-108MHz. The IF and local
oscillator frequencies are chosen such that f IF <f LO . We require that the image frequency
f ℑfall outside the range of 88-108MHz. Determine the minimum required f IF and the
range of variations in f LO .
a. Determine hl ( t) the low pass equivalent of h ( t ) . Where h ( t ) represents the impulse
response of the BPF.
b. Derive an expression for the modulated signal.

25. Consider a super heterodyne receiver designed to receive a frequency band of 1-30MHz
with IF frequency of 8MHz. What is the range of frequencies generated by local
oscillator for this receiver? An incoming signal with carrier frequency 10MHz is received
at 10MHz setting. At this setting of the receiver we also get interference from the signal
with some other carrier frequency if the receiver RF stage BPF has poor selectivity. What
is the carrier frequency of the interfering signal?
26. Sketch FM and PM waves for the modulating signal shown in figure 5.4a. The constants
k f ∧k p are 2 π ×10 5∧10 π respectively and the carrier frequency f c =100 MHz .

a. Estimate the bandwidth of FM and PM for modulating signal shown in Figure


5.4a. for k f =2 π ×105∧k p =5 π
b. Repeat part a. if amplitude of m(t) is doubled.
c. Repeat part a. if m(t) is time expanded by factor of 2.
27. A baseband signal m ( t ) is the periodic sawtooth signal shown in figure P.5.1-2. Sketch
6 π
the modulated FM and PM waves for this m ( t ) if ω c =2 π ×10 , k f =2000 π∧k p= .
2
Explain why it is necessary to use k p< π in this case. Assume the bandwidth of m(t) to be
fifth harmonic frequency and estimate the bandwidth for PM and FM.
28. Over an interval |t |≤1 , an angle modulated signal is given by φ EM (t )=10 cos 13000 t . It is
known that carrier frequency ω c =10,000.
a. If this were a PM signal, with k p=1000 , determine m ( t ) over the interval |t |≤1.
b. If this were a FM signal, with k f =1000 , determine m ( t ) over the interval |t |≤1.
29. For modulating signal m ( t )=2cos 100 t +18 cos 2000 πt
a. Write the expressions for PM and FM for
A=10 , ωc =106 , k f =1000 π∧k p=1.
b. Estimate the bandwidth for FM and PM.
30. An angle modulated signal with carrier frequency ω c =2 π ×10 6 is described by the
equation φ EM (t )=5 cos ¿ ¿. Find the
e. Power of the angle modulated signal.
f. Frequency deviation ∆ f
g. Phase deviation ∆ φ .
h. Estimate the bandwidth of φ EM (t )
31. A carrier signal is frequency modulated by a message signal m ( t )= A m cos 2 π f m t . In a
certain experiment conducted with f m=1 KHz increasing Am (starting from 0V) it was
found that the carrier magnitude in a spectrum becomes zero for the first time when
Am =2V .
a. Determine the frequency sensitivity of the modulator.
b. Determine the value of AM for which the carrier magnitude becomes zero for the
second time.
32. FM transmitter radiates 100W when the carrier is not modulated. The carrier is now
modulated and modulation index is adjusted so that magnitude of 1 st order sidebands is
zero in the spectrum. Under this conditions,
a. Calculate the power at carrier frequency
b. Calculate the power in all remaining sidebands
c. Calculate the power in second order sidebands.
Given J 0 ( 0 )=1 , J 0 ( 2.4 )=0 , J 0 ( 3.8 )=−0.4 , J 0 (5.1 ) =−0.16 ;

J 1 ( 2.4 )=0.52, J 1 (3.8 )=0 , J 1 ( 5.1 ) =−0.33;

J 2 ( 2.4 )=0.43 , J 2 ( 3.8 )=0.41 , J 2 ( 5.1 ) =0

33. A carrier is frequency modulated by a sinusoidal signal and frequency deviation is


50KHz. Determine the modulation index and bandwidth when
a. f m=500 kHz
b. f m=500 Hz
34. Given that m ( t )=sin2000 πt , k f =200,000 π , k p=10.
a. Estimate the bandwidths of FM and PM.
b. Repeat part a. if message signal amplitude is doubled
c. Repeat part a. if message signal frequency is doubled.
d. Comment on the sensitivity of PM and FM bandwidths to the spectrum of m ( t )
35. A 107.6 MHz carrier signal is frequency modulated by a 7 kHz sine wave. The resultant
FM signal has frequency deviation of 50 kHz. Determine the following:
a. The carrier swing of the FM signal
b. The highest and the lowest frequencies attained by the modulated signal.
c. The modulation index of the FM wave.
36. Find the bandwidth of commercial FM transmission assuming frequency deviation
∆ f =75 kHz and bandwidth of modulating signal f m=15 kHz

Вам также может понравиться